Bộ sưu tập bất đẳng thức của Võ Quốc Bá Cẩn

65 1.7K 6
Bộ sưu tập bất đẳng thức của Võ Quốc Bá Cẩn

Đang tải... (xem toàn văn)

Tài liệu hạn chế xem trước, để xem đầy đủ mời bạn chọn Tải xuống

Thông tin tài liệu

u›.?p·›Ž¦?a\2?b\ž‹ `‹?h‹¡fl·\„?¦›¡¦›‹ k¡?⁄¡?›·›‹?\„?„›·‒?«¡⁄›@ s⁄¡ ¡¦›‹?¡‒›‹ b\Ÿ‹?s⁄-?Å?QOOX Võ Quốc Cẩn Copyright c  2009 by Vo Quoc Ba Can. All rights reserved. No part of this book may be reproduced or distributed in any form or by any means, or stored in data base or a retrieval system, without the prior written the permission of the author. Lời cảm ơn Quyển tuyển tập này chắc chắn sẽ không thể thực hiện được nếu không có sự đóng góp của những người bạn của tôi. Họ đã trực tiếp động viên tôi thực hiện, gửi cho tôi những bài toán hay giúp tôi có thể tuyển tập lại một cách tốt nhất có thể các bài toán bất đẳng thức. Xin được nêu ra đây những người bạn thân thiết đã giúp đỡ tôi rất nhiều trong quá trình thực hiện quyển tuyển tập này 1. Nguyễn Văn Dũng - Giảng viên Học Viện Kỹ Thuật Quân Sự Hà Nội. 2. Trần Quang Hùng - Cao học toán trường Đại Học Khoa Học Tự Nhiên, ĐHQG Hà Nội. 3. Cao Minh Quang - Giáo viên trường THPT Chuyên Nguyễn Bỉnh Khiêm, Vĩnh Long. 4. Thành Văn - Lớp 12 Toán, trường THPT Chuyên, ĐHKH Huế. 5. Nguyễn Mạnh Dũng - Lớp 12 Toán, khối Phổ Thông Chuyên Toán – Tin, trường ĐHKHTN, ĐHQH Hà Nội. 6. Trần Anh Tuấn - đang cập nhật thông tin. Những bài bất đẳng thức từ các cuộc thi giải toán Bài O1. Giả sử a,b,c là các số thực không âm thỏa mãn a 2 + b 2 + c 2 + abc = 4. Chứng minh rằng 0 ≤ab + bc+ ca −abc ≤ 2. (USAMO 2000) Lời giải 1 (V. Q. B. Cẩn). Bất đẳng thức bên trái là hiển nhiên, bởi vì từ giả thiết, ta suy ra có ít nhất một số trong ba số a,b,c không lớn hơn 1. Giả sử số đó là c, khi đó ta sẽ có ab + bc+ ca −abc = ab(1 −c) +c(a + b) ≥0. Bây giờ, ta sẽ chứng minh bất đẳng thức bên phải. Thay abc = 4 −(a 2 + b 2 + c 2 ) vào, ta có thể viết lại bất đẳng thức này thành a 2 + b 2 + c 2 + ab + bc + ca ≤6. Ta sẽ dùng phương pháp phản chứng để chứng minh bất đẳng thức này. Giả sử tồn tại một bộ số (a,b,c) gồm các số hạng không âm sao cho a 2 + b 2 + c 2 + abc = 4 và a 2 + b 2 + c 2 + ab + bc + ca > 6. Khi đó, ta sẽ có 4 = a 2 + b 2 + c 2 + abc = 6(a 2 + b 2 + c 2 ) 6 + 6 √ 6abc 6 √ 6 > 6(a 2 + b 2 + c 2 ) a 2 + b 2 + c 2 + ab + bc + ca + 6 √ 6abc (a 2 + b 2 + c 2 + ab + bc + ca) 3/2 , suy ra 2(ab + bc + ca) −(a 2 + b 2 + c 2 ) > 3 √ 6abc √ a 2 + b 2 + c 2 + ab + bc + ca . Mặt khác, áp dụng bất đẳng thức Schur bậc 4 (ở dạng phân thức), ta thấy 2(ab + bc + ca) −(a 2 + b 2 + c 2 ) ≤ 6abc(a + b + c) a 2 + b 2 + c 2 + ab + bc + ca , nên từ trên ta suy ra 6abc(a + b + c) a 2 + b 2 + c 2 + ab + bc + ca > 3 √ 6abc √ a 2 + b 2 + c 2 + ab + bc + ca . Điều này chứng tỏ rằng abc > 0 và √ 2(a + b + c) >  3(a 2 + b 2 + c 2 + ab + bc + ca). Điều này vô lí, bởi vì ta luôn có 3(a 2 + b 2 + c 2 + ab + bc + ca) −2(a + b + c) 2 = a 2 + b 2 + c 2 −ab −bc −ca ≥0. Như vậy, không thể nào tồn tại các số a,b,c thỏa mãn giả thiết của đề bài sao cho a 2 +b 2 +c 2 +ab + bc + ca > 6, hay nói một cách khác, với mọi a,b,c không âm sao cho a 2 + b 2 + c 2 + abc = 4, ta phải có ab + bc + ca −abc ≤2. Bài toán được chứng minh xong. Dễ thấy bất đẳng thức bên trái đạt được dấu bằng khi (a,b,c) là một hoán vị của bộ số (2, 0,0); và bất đẳng thức bên phải đạt được dấu bằng khi (a,b,c) = (1,1,1) hoặc (a,b,c) là một hoán vị của bộ số  √ 2, √ 2,0  . Những bài bất đẳng thức từ các cuộc thi giải toán 5 Lời giải 2. Đây là một chứng minh rất hay và đặc sắc cho bất đẳng thức bên phải. Trong ba số a,b,c, luôn tồn tại ít nhất 2 số sao cho hiệu của chúng khi trừ cho 1 có cùng dấu với nhau. Không mất tính tổng quát, giả sử hai số đó là a và b, khi đó ta có c(a −1)(b −1) ≥0, suy ra abc ≥ac + bc −c. Mặt khác, theo bất đẳng thức AM – GM thì 4 = a 2 + b 2 + c 2 + abc ≥ 2ab + c 2 + abc, suy ra ab ≤ 2 −c. Từ đây, ta thu được ab + bc + ca −abc ≤(2 −c) + bc + ca −(ac + bc −c) = 2. Lời giải 3 (V. Q. B. Cẩn). Xin được giới thiệu thêm cùng bạn đọc một chứng minh khác cho bất đẳng thức bên phải. Từ giả thiết, ta dễ dàng chứng minh được tồn tại các số không âm x,y, z sao cho (x + y)(y + z)(z + x) > 0 và a = 2x √ (x+y)(x+z) ,b = 2y √ (y+z)(y+x) ,c = 2z √ (z+x)(z+y) . Với phép đặt thuần nhất này, ta có thể đưa bài toán về chứng minh 2 ∑ cyc xy (x + y)  (x + z)(y + z) − 4xyz (x + y)(y + z)(z + x) ≤ 1. Áp dụng bất đẳng thức AM – GM, ta có 2 ∑ cyc xy (x + y)  (x + z)(y + z) ≤ ∑ cyc xy x + y  1 x + z + 1 y + z  = ∑ cyc xy (x + y)(x + z) + ∑ cyc xy (y + z)(y + x) = ∑ cyc xy (x + y)(x + z) + ∑ cyc zx (x + y)(x + z) = ∑ cyc x(y + z) (x + y)(x + z) = 1+ 4xyz (x + y)(y + z)(z + x) . Vì thế bất đẳng thức trên là hiển nhiên đúng, và phép chứng minh của ta được hoàn tất. Bài O2. Cho a,b,c là các số thực dương thỏa mãn ab + bc + ca + abc = 4. Chứng minh rằng a + b + c ≥ab + bc + ca. (Việt Nam, 1996) Lời giải 1 (V. Q. B. Cẩn). Từ giả thiết, suy ra ta có thể đặt a = 2x y+z ,b = 2y z+x và c = 2z x+y với x,y,z là các số thực dương. Khi đó, bất đẳng thức cần chứng minh có thể được viết lại thành x y + z + y z + x + z x + y ≥ 2xy (x + z)(y + z) + 2yz (y + x)(z + x) + 2zx (z + y)(x + y) . Áp dụng bất đẳng thức AM – GM, ta có V P ≤ ∑ cyc xy  1 (x + z) 2 + 1 (y + z) 2  = ∑ cyc xy (z + x) 2 + ∑ cyc xy (y + z) 2 = ∑ cyc zx (y + z) 2 + ∑ cyc xy (y + z) 2 = ∑ cyc x y + z = VT. Phép chứng minh của ta được hoàn tất. Dễ thấy đẳng thức xảy ra khi và chỉ khi x = y = z, tức là a = b = c = 1. 6 Let the solutions say your method - Quốc Cẩn Lời giải 2 (V. Q. B. Cẩn). Ta sẽ dùng phương pháp ph ản chứng. Giả sử rằng tồn tại các số dương a,b,c sao cho ab + bc + ca + abc = 4 và a + b + c < ab + bc + ca. Khi đó, ta có a+b+c ab+bc+ca < 1, dẫn đến 4 = (ab + bc + ca) ·1 + abc ·1 > (ab + bc + ca) ·  a + b + c ab + bc + ca  2 + abc ·  a + b + c ab + bc + ca  3 = (a + b + c) 2 ab + bc + ca + abc(a + b + c) 3 (ab + bc + ca) 3 . Từ đây, ta tìm được 2(ab + bc + ca) −(a 2 + b 2 + c 2 ) > abc(a + b + c) 3 (ab + bc + ca) 2 . Nhưng mà theo bất đẳng thức Schur bậc 3 ở dạng phân thức thì 2(ab + bc + ca) −(a 2 + b 2 + c 2 ) ≤ 9abc a+b+c . Điều này dẫn đến 9abc a + b + c > abc(a + b + c) 3 (ab + bc + ca) 2 , suy ra abc > 0 và 9(ab + bc + ca) 2 > (a + b + c) 4 (mâu thuẫn bởi vì ta luôn có (a + b + c) 2 ≥ 3(ab + bc + ca) theo AM – GM). Bởi vậy, ta không thể có a +b + c < ab +bc + ca với mọi a, b,c > 0 thỏa mãn giả thiết của đề bài. Điều này chứng tỏ rằng a+b+c ≥ab+bc+ca, đây chính là điều phải chứng minh. Lời giải 3 (V. Q. B. Cẩn). Ta sẽ sử dụng phương pháp dồn biến để chứng minh bất đẳng thức đã cho. Để ý rằng ngoài điểm đẳng thức là a = b = c = 1 thì bất đẳng thức đã cho còn có một điểm "nhạy cảm" là a = b → 2, c → 0 (cùng các hoán vị). Điều này gợi cho ta giả sử c = min{a,b,c} và dùng phép dồn biến để đưa hai biến a,b về bằng nhau và bằng một số t dương nào đó. Muốn vậy, việc trước tiên ta phải làm đó là đảm bảo giả thiết của bài toán, tức là bộ số (t,t,c) phải thỏa mãn t 2 + 2tc + t 2 c = ab + bc + ca + abc = 4. Vì ta cần dồn biến từ (a,b,c) về (t,t,c) nên ta phải chứng minh a + b + c −ab −bc −ca ≥2t + c −t 2 −2tc, tương đương (a + b −2t)(1 −c) + (t 2 −ab) ≥ 0. (∗) Mặt khác, từ cách chọn của t, ta có c(a + b −2t) = (c + 1)(t 2 −ab). Ta sẽ chứng minh a + b −2t và t 2 −ab là những số không âm. Thật vậy, giả sử a + b −2t < 0, khi đó ta cũng có t 2 −ab < 0. Điều này dẫn đến ab > t 2 > (a+b) 2 4 ≥ ab (vô lí). Vì vậy, ta phải có a + b −2t ≥0 và t 2 −ab ≥ 0. Ngoài ra, từ giả thiết của c, dễ thấy c ≤1. Và như thế, bất đẳng thức (∗) là hiển nhiên đúng. Phép dồn biến đã được hoàn tất, công việc còn lại của ta chỉ là chứng minh 2t + c −t 2 −2tc ≥0 với t 2 + 2tc +t 2 c = 4. Đây là một công việc rất đơn giản, bởi vì từ t 2 + 2tc +t 2 c = 4, ta tìm được c = 2−t t ≥ 0, dẫn đến 2t + c −t 2 −2tc = 2t + 2 −t t −t 2 −2(2 −t) = (2 −t)(t −1) 2 t ≥ 0. Lời giải 4 (V. Q. B. Cẩn). Dễ thấy rằng trong ba số a,b,c có ít nhất hai số có hiệu khi trừ cho 1 là những số cùn g dấu với nhau. Giả sử hai số đó là a, b, khi đó ta sẽ có c(a −1)(b −1) ≥ 0, dẫn đến abc ≥ac + bc −c. Từ đây, ta thu được a + b + c + abc ≥(a + b)(c + 1). Những bài bất đẳng thức từ các cuộc thi giải toán 7 Mặt khác, áp dụng bất đẳng thức AM – GM, ta lại có 4 = abc + c(a + b) + ab ≤ (a + b) 2 4 ·c + c(a + b) + (a + b) 2 4 , suy ra c ≥ 4 − (a+b) 2 4 (a+b) 2 4 + (a + b) = 4 −(a + b) a + b = 4 a + b −1. Cộng 1 vào hai vế của bất đẳng thức này rồi nhân cho a + b > 0, ta thu được ngay (a + b)(c + 1) ≥4. Do đó, kết hợp với trên, ta được a + b + c + abc ≥ (a + b)(c + 1) ≥ 4 = ab + bc + ca + abc, hay nói một cách khác a + b + c ≥ab + bc + ca. Bài O3. Với a,b,c là các số thực dương bất kì, hãy tìm tất cả các số thực k để cho bất đẳng thức sau đúng  k + a b + c  k + b c + a  k + c a + b  ≥  k + 1 2  3 . (Việt Nam, 2009) Lời giải (V. Q. B. Cẩn). Đầu tiên, ta cho a = b = 1, bất đẳng thức đã cho trở thành  k + 1 1+c  2  k + c 2  ≥  k + 1 2  3 , tương đương (c −1) 2 (4k 2 c + 4k 2 + 2k −1) 8(c + 1) 2 ≥ 0. Đến đây, cho c → 0, ta thấy bất đẳng thức chỉ đúng nếu 4k 2 + 2k −1 ≥ 0. Ta sẽ chứng minh rằng, nghiệm của bất phương trình này chính là tập hợp tất cả các giá trị của k thỏa mãn yêu cầu bài toán, tức là chứng minh với 4k 2 + 2k −1 ≥0 thì  k + a b + c  k + b c + a  k + c a + b  ≥  k + 1 2  3 . Thật vậy, đặt x = 2a b+c ,y = 2b c+a ,z = 2c a+b thì hiển nhiên xy + yz + zx + xyz = 4 và bất đẳng thức trên được viết lại thành (2k + x)(2k + y)(2k +z) ≥ (2k + 1) 3 . Bây giờ, áp dụng bất đẳng thức AM – GM, ta dễ thấy xyz ≤1. Từ đó, sử dụng kết quả bài O2, ta thu được (2k + x)(2k + y)(2k + z) = 8k 3 + 4k 2 (x + y + z) + 2k(xy + yz + zx) + xyz ≥ 8k 3 + 4k 2 (xy + yz + zx) + 2k(xy + yz + zx) + xyz = 8k 3 + (4k 2 + 2k)(4 −xyz) + xyz = 8k 3 + 16k 2 + 8k −(4k 2 + 2k −1)xyz ≥ 8k 3 + 16k 2 + 8k −(4k 2 + 2k −1) = (2k + 1) 3 . Như vậy, phép chứng minh của ta đã được hoàn tất. Điều này cũng chứng tỏ rằng khẳng định của ta ở trên là đúng, tức là tập hợp tất cả các giá trị cần tìm của k chính là nghiệm của bất phương trình 4k 2 + 2k −1 ≥0. Bài O4. Cho a,b,c,d là các số thực dương thỏa mãn 1 a 4 + 1 + 1 b 4 + 1 + 1 c 4 + 1 + 1 d 4 + 1 = 1. 8 Let the solutions say your method - Quốc Cẩn Chứng minh rằng abcd ≥3. (Latvia 2002) Lời giải 1 (V. Q. B. Cẩn). Áp dụng bất đẳng thức Cauchy Schwarz, ta có 1 = 1 a 4 + 1 + 1 b 4 + 1 + 1 c 4 + 1 + 1 d 4 + 1 = 1 a 4 1 a 4 + 1 + 1 b 4 1 b 4 + 1 + 1 c 4 1 c 4 + 1 + 1 d 4 1 d 4 + 1 ≥  1 a 2 + 1 b 2 + 1 c 2 + 1 d 2  2 1 a 4 + 1 b 4 + 1 c 4 + 1 d 4 + 4 . Từ đó suy ra 1 a 4 + 1 b 4 + 1 c 4 + 1 d 4 + 4 ≥  1 a 2 + 1 b 2 + 1 c 2 + 1 d 2  2 , tức là 2 ≥ 1 a 2 b 2 + 1 a 2 c 2 + 1 a 2 d 2 + 1 b 2 c 2 + 1 b 2 d 2 + 1 c 2 d 2 . Mà theo bất đẳng thức AM – GM thì 1 a 2 b 2 + 1 a 2 c 2 + 1 a 2 d 2 + 1 b 2 c 2 + 1 b 2 d 2 + 1 c 2 d 2 ≥ 6 abcd nên kết hợp với trên, ta dễ dàng suy ra được bất đẳng thức cần chứng minh. Đẳng thức xảy ra khi và chỉ khi a = b = c = d = 4 √ 3. Lời giải 2. Đặt x = 1 a 4 +1 ,y = 1 b 4 +1 ,z = 1 c 4 +1 và t = 1 d 4 +1 thì ta có x + y + z + t = 1 và a 4 = 1 −x x = y + z +t x , b 4 = z + t +x y , c 4 = t + x + y z , d 4 = x + y + z t . Từ đó, để chứng minh bất đẳng thức abcd ≥ 3, ta thấy rằng ta chỉ cần chứng minh được y + z +t x · z + t +x y · t + x + y z · x + y + z t ≥ 81. Nhưng bất đẳng thức này hiển nhiên đúng bởi vì theo AM – GM, ta có y + z +t x · z + t +x y · t + x + y z · x + y + z t ≥ 3 3 √ yzt x · 3 3 √ ztx y · 3 3 √ txy z · 3 3 √ xyz t = 81. Phép chứng minh của ta được hoàn tất. Bài O5. Cho các số dương a,b,c thỏa mãn 1 a + b + 1 + 1 b + c + 1 + 1 c + a + 1 ≥ 1. Chứng minh rằng a + b + c ≥ab + bc + ca. (Andrei Ciupan, Chọn đội tuyển Romania dự thi Junior BMO 2007) Lời giải 1 (Andrei Ciupan). Áp dụng bất đẳng thức Cauchy Schwarz, dễ thấy (a + b + 1)(a + b + c 2 ) ≥(a + b + c) 2 . Từ đó dẫn đến 1 ≤ 1 a + b + 1 + 1 b + c + 1 + 1 c + a + 1 ≤ a + b + c 2 (a + b + c) 2 + b + c + a 2 (a + b + c) 2 + c + a + b 2 (a + b + c) 2 , suy ra (a + b + c) 2 ≤ 2(a + b + c) + a 2 + b 2 + c 2 , Những bài bất đẳng thức từ các cuộc thi giải toán 9 tức là a + b + c ≥ab + bc + ca. Bất đẳng thức của ta được chứng minh xong. Đẳng thức xảy ra khi và chỉ khi a = b = c = 1. Lời giải 2 (Cezar Lupu). Từ giả thiết, sử dụng bất đẳng thức Cauchy Schwarz, ta có 2 ≥  1 − 1 a + b + 1  +  1 − 1 b + c + 1  +  1 − 1 c + a + 1  = a + b a + b + 1 + b + c b + c + 1 + c + a c + a + 1 ≥ [(a + b) + (b + c) + (c + a)] 2 (a + b)(a + b + 1) + (b + c)(b + c + 1) + (c + a)(c + a + 1) = 2(a 2 + b 2 + c 2 ) + 4(ab + bc + ca) (a 2 + b 2 + c 2 ) + (ab + bc + ca) + (a + b + c) . Từ đây, ta suy ra được (a 2 + b 2 + c 2 ) + (ab + bc + ca) + (a + b + c) ≥(a 2 + b 2 + c 2 ) + 2(ab + bc + ca), tức là a + b + c ≥ab + bc + ca. Đây chính là điều phải chứng minh. Lời giải 3 (V. Q. B. Cẩn). Ta sẽ dùng phương pháp phản chứng để chứng minh bất đẳng thức này. Giả sử tồn tại các số dương a,b,c sao cho 1 a+b+1 + 1 b+c+1 + 1 c+a+1 ≥ 1 và a + b + c < ab + bc + ca. Khi đó, ta có 1 < ab+bc+ca a+b+c , dẫn đến 1 a + b + 1 < ab+bc+ca a+b+c a + b + ab+bc+ca a+b+c = ab + bc + ca (a + b)(a + b + c) + ab + bc + ca . Và ta thu được ∑ cyc ab + bc + ca (a + b)(a + b + c) + ab + bc + ca > 1, tương đương 1 > ∑ cyc  1 − 2(ab + bc + ca) (a + b)(a + b + c) + ab + bc + ca  , hay là 1 > ∑ cyc a 2 + ab + b 2 (a + b)(a + b + c) + ab + bc + ca . Tuy nhiên, theo các bất đẳng thức AM – GM và Cauchy Schwarz thì V P ≥ 3 4 ∑ cyc (a + b) 2 (a + b)(a + b + c) + ab + bc + ca ≥ 3(a + b + c) 2 ∑ cyc [(a + b)(a + b + c) + ab + bc + ca] = 3(a + b + c) 2 2(a + b + c) 2 + 3(ab + bc + ca) ≥ 3(a + b + c) 2 2(a + b + c) 2 + (a + b + c) 2 = 1 (mâu thuẫn). Vì vậy, ta không thể có điều giả sử trên, tức là với mọi a, b, c dương thỏa mãn 1 a+b+1 + 1 b+c+1 + 1 c+a+1 ≥ 1 thì bắt buộc ta phải có a + b + c ≥ab + bc + ca. Phép chứng minh được hoàn tất. 10 Let the solutions say your method - Quốc Cẩn Bài O6. Cho n ≥2 là một số nguyên bất kì. Tìm hằng số C nhỏ nhất để bất đẳng thức sau ∑ 1≤i< j≤n x i x j (x 2 i + x 2 j ) ≤C(x 1 + x 2 + ···+ x n ) 4 , luôn đúng với mọi số thực không âm x 1 ,x 2 , ,x n . (IMO 1999) Lời giải (V. Q. B. Cẩn). Với n = 2, cho x 1 = x 2 = 1, ta dễ thấy C ≥ 1 8 . Xét trường hợp n ≥ 3, cho x 1 = x 2 = 1,x 3 = ···= x n = 0, ta cũng tìm được C ≥ 1 8 . Ta sẽ chứng minh rằng 1 8 cũng chính là giá trị nhỏ nhất của C để bất đẳng thức trên đúng, tức là ∑ 1≤i< j≤n x i x j (x 2 i + x 2 j ) ≤ 1 8 (x 1 + x 2 + ···+ x n ) 4 . Thật vậy, áp dụng bất đẳng thức AM – GM, ta có ∑ 1≤i< j≤n x i x j (x 2 i + x 2 j ) ≤ ∑ 1≤i< j≤n x i x j  x 2 i + x 2 j + ∑ k=i,k= j x 2 k  =  ∑ 1≤i< j≤n x i x j  n ∑ i=1 x 2 i  = 1 2 ·  2 ∑ 1≤i< j≤n x i x j  ·  n ∑ i=1 x 2 i  ≤ 1 2     2 ∑ 1≤i< j≤n x i x j + n ∑ i=1 x 2 i 2     2 = 1 8  n ∑ i=1 x i  4 . Như thế, khẳng định của ta đã được chứng minh xong. Điều này cho phép ta đi đến kết luận hằng số C nhỏ nhất thỏa mãn yêu cầu của đề bài là C min = 1 8 . Bài O7. Chứng minh rằng với mọi số thực dương a,b,c,x,y,z, bất đẳng thức sau luôn được thỏa mãn ax a + x + by b + y + cz c + z ≤ (a + b + c)(x + y + z) a + b + c + x + y + z . (KMO Weekend Program 2007) Lời giải 1 (V. Q. B. Cẩn). Bất đẳng thức cần chứng minh có thể được viết lại như sau  a + x 4 − ax a + x  +  b + y 4 − by b + y  +  c + z 4 − cz c + z  ≥ a + b + c + x + y + z 4 − (a + b + c)(x + y + z) a + b + c + x + y + z , hay là (a −x) 2 a + x + (b −y) 2 b + y + (c −z) 2 c + z ≥ (a + b + c −x −y −z) 2 a + b + c + x + y + z . Theo bất đẳng thức Cauchy Schwarz, ta dễ thấy V T ≥ [(a −x) + (b −y) + (c −z)] 2 (a + x) + (b + y) + (c + z) = VP, và như thế, bất đẳng thức của ta đã được chứng minh xong. [...]... trường hợp ta đều có n2 n−1 Đẳng thức xảy ra khi và chỉ khi x1 = x2 = · · · = xn = n2 n−1 n n−1 Những bài bất đẳng thức tự sáng tạo và sưu tầm Bài CH1 Giả sử a, b, c là độ dài ba cạnh của một tam giác, hãy chứng minh bất đẳng thức sau bc ca ab 3 + + ≤ a4 + 2b2 c2 b4 + 2c2 a2 c4 + 2a2 b2 a2 + b2 + c2 (Võ Quốc Cẩn) Lời giải (V Q B Cẩn) Nhân cả hai vế của bất đẳng thức đã cho với a2 b2 + b2 c2... rằng bất đẳng thức này chính là tổng của hai bất đẳng thức sau c a b + + b c a 2 ≥ (a + b + c) và c a b + + b c a 1 1 1 + + , a b c 2 ≥ (a + b + c)2 36 Let the solutions say your method - Võ Quốc Cẩn Bất đẳng thức thứ nhất tương đương với b2 c2 a2 a b c b c a + + + + + ≥ + + + 3, a2 b 2 c 2 b c a a b c 2 2 2 2 b c c c c mà a2 + b2 + a2 ≥ 1 b + b + a ≥ b + b + a và a + b + a ≥ 3 nên bất đẳng thức. .. bài toán và bất đẳng thức trung bình lũy thừa, ta có a + b = a4 + b4 ≥ (a+b) , 8 suy ra a + b ≤ 2, mà a, b là các số dương nên a, b ∈ (0, 2) Vì thế, áp dụng bất đẳng thức vừa chứng minh, ta có a4 − a b4 − b 3(a3 ln a + b3 ln b) ≥ a3 · + b3 · = a4 + b4 − a − b = 0 a3 b3 31 Những bài bất đẳng thức tự sáng tạo và sưu tầm Bất đẳng thức bên phải được chứng minh xong Dễ thấy ở cả hai bất đẳng thức (bên trái... a+b+c+d Cộng tương ứng vế với vế hai bất đẳng thức này, ta dễ dàng thu được bất đẳng thức cần chứng minh Đẳng thức xảy ra khi và chỉ khi a = c và b = d Bài O20 Cho a, b, c là các số thực dương có tổng bằng 3 Chứng minh bất đẳng thức sau a b c 3 + + ≥ b + c2 c + a2 a + b2 2 (Phạm Kim Hùng, Tập huấn đội tuyển Việt Nam dự thi IMO 2009) Lời giải (V Q B Cẩn) Sử dụng bất đẳng thức Cauchy Schwarz, ta dễ thấy... được chứng minh xong Bài O15 Chứng minh rằng với mọi 0 < x < π , bất đẳng thức sau luôn được thỏa mãn 4 (cos x)cos x > (sin x)sin x 18 Let the solutions say your method - Quốc Cẩn (MOSP 2004) Lời giải (V Q B Cẩn) Ta viết lại bất đẳng thức cần chứng minh dưới dạng sau (cos x)cot x > sin x, hay là (cos2 x)cot x > sin2 x Áp dụng bất đẳng thức Bernoulli với để ý rằng cot x > 1 ∀x ∈ 0, π , ta được 4... + c) 2 2 2 Áp dụng bất đẳng thức Cauchy Schwarz, ta có (a−b) + (b−c) ≥ (a−c) Do đó, ta chỉ cần chứng minh b c b+c được 1 a+b+c 1 + ≥ b + c a (a + b)(b + c) là một bất đẳng thức hiển nhiên đúng bởi vì nó tương đương với b(a + b + c) ≥ 0 a(a + b)(b + c) Phép chứng minh của ta được hoàn tất Đẳng thức xảy ra khi và chỉ khi a = b = c 12 Let the solutions say your method - Quốc Cẩn Lời giải 2 Đặt... b)2 + (2ab − 1)2 ≥ 0, nên bất đẳng thức trên hiển nhiên đúng Bài toán được chứng minh xong Dễ thấy đẳng thức xảy ra khi và chỉ khi ui = vi và u2 + u2 + · · · + u2 = 1 n 2 1 2 20 Let the solutions say your method - Quốc Cẩn Bài O19 Chứng minh rằng với mọi a, b, c, d dương, ta đều có a+c b+d c+a d +b + + + ≥ 4 a+b b+c c+d d +a (Dự tuyển IMO 1971) Lời giải Áp dụng bất đẳng thức AM – GM, ta có 1 1... viết lại bất đẳng thức 2(b + c − a)2 2(c + a − b)2 2(a + b − c)2 + 2 + 2 ≥ 1 2a2 + (b + c)2 2b + (c + a)2 2c + (a + b)2 Mà theo bất đẳng thức Cauchy Schwarz thì 2(b + c − a)2 (b + c − a)2 2(b + c − a)2 ≥ 2 = 2 2a2 + (b + c)2 2a + 2(b2 + c2 ) a + b2 + c2 Do đó, ta chỉ cần chứng minh được (b + c − a)2 + (c + a − b)2 + (a + b − c)2 ≥ a2 + b2 + c2 2 2 Bất đẳng thức này được suy ra từ bất đẳng thức sau... 1 và 2(a + c + d) ≥ a + 2c nên bất đẳng thức này là hiển nhiên Nếu a+b+c b+c+d ≥ a+c c+d nên bất đẳng thức trên là hệ quả của √ √ 2(a + c + d) a + c ≥ (a + 2c) c + d Ta có √ √ √ 2(a + c + d) a + c = 2 a + c + d (a + c + d)(a + c) ≥ 2(a + c) a + c + d √ √ ≥ 2(a + c) c + d ≥ (a + 2c) c + d, nên bất đẳng thức cuối hiển nhiên đúng Bài toán được chứng minh xong Dễ thấy đẳng thức xảy ra khi và chỉ khi a =... c2 2 2 Bất đẳng thức này được suy ra từ bất đẳng thức sau (b+c−a) +(c+a−b) ≥ c2 (đúng theo Cauchy 2 Schwarz) và hai bất đẳng thức tương tự Như vậy, bài toán của ta đã được chứng minh xong Dễ thấy đẳng thức xảy ra khi và chỉ khi a = b = c Lời giải 2 Bất đẳng thức đã cho là một bất đẳng thức thuần nhất bậc 0 Vì thế, ta có thể chuẩn hóa cho a + b + c = 1, khi đó, nó được viết lại thành (b + 1)2 (c + 1)2 . c 2 , Những bài bất đẳng thức từ các cuộc thi giải toán 9 tức là a + b + c ≥ab + bc + ca. Bất đẳng thức của ta được chứng minh xong. Đẳng thức xảy ra khi. a 2 + b 2 + c 2 . Bất đẳng thức này được suy ra từ bất đẳng thức sau (b+c−a) 2 +(c+a−b) 2 2 ≥ c 2 (đúng theo Cauchy Schwarz) và hai bất đẳng thức tương tự.

Ngày đăng: 21/03/2014, 19:31

Từ khóa liên quan

Tài liệu cùng người dùng

Tài liệu liên quan